Last visit was: 23 Apr 2024, 13:14 It is currently 23 Apr 2024, 13:14

Close
GMAT Club Daily Prep
Thank you for using the timer - this advanced tool can estimate your performance and suggest more practice questions. We have subscribed you to Daily Prep Questions via email.

Customized
for You

we will pick new questions that match your level based on your Timer History

Track
Your Progress

every week, we’ll send you an estimated GMAT score based on your performance

Practice
Pays

we will pick new questions that match your level based on your Timer History
Not interested in getting valuable practice questions and articles delivered to your email? No problem, unsubscribe here.
Close
Request Expert Reply
Confirm Cancel
SORT BY:
Date
Tags:
Show Tags
Hide Tags
avatar
Intern
Intern
Joined: 15 Jan 2013
Posts: 40
Own Kudos [?]: 84 [30]
Given Kudos: 10
Send PM
Most Helpful Reply
User avatar
Manager
Manager
Joined: 21 Aug 2014
Posts: 104
Own Kudos [?]: 806 [7]
Given Kudos: 49
GMAT 1: 610 Q49 V25
GMAT 2: 730 Q50 V40
Send PM
General Discussion
User avatar
Senior Manager
Senior Manager
Joined: 03 Aug 2011
Posts: 267
Own Kudos [?]: 106 [1]
Given Kudos: 916
Concentration: Strategy, Finance
GMAT 1: 640 Q44 V34
GMAT 2: 700 Q42 V44
GMAT 3: 680 Q44 V39
GMAT 4: 740 Q49 V41
GPA: 3.7
WE:Project Management (Energy and Utilities)
Send PM
Senior Manager
Senior Manager
Joined: 27 Mar 2017
Posts: 274
Own Kudos [?]: 76 [0]
Given Kudos: 406
Location: Saudi Arabia
GMAT 1: 700 Q47 V39
GPA: 3.36
Send PM
Re: The Orange corporation is conducting market research in preparation fo [#permalink]
Can someone comment on my train of thought that made me exclude A and arrive at D as the correct answer.

The argument states that 25% of group A (docs) + group B (>$250k) are expected to account for the sales. It does not state any where how many of individual groups. So, even if the statement A is true, the remaining 20% can be provided by the group B guys(there is no hint in the argument why it can't be so).

Is this a correct approach ? I don't want to arrive at a correct answer with wrong logic.

Thanks.

Posted from my mobile device
GMAT Club Legend
GMAT Club Legend
Joined: 03 Oct 2013
Affiliations: CrackVerbal
Posts: 4946
Own Kudos [?]: 7624 [0]
Given Kudos: 215
Location: India
Send PM
Re: The Orange corporation is conducting market research in preparation fo [#permalink]
Top Contributor
altairahmad wrote:
Can someone comment on my train of thought that made me exclude A and arrive at D as the correct answer.

The argument states that 25% of group A (docs) + group B (>$250k) are expected to account for the sales. It does not state any where how many of individual groups. So, even if the statement A is true, the remaining 20% can be provided by the group B guys(there is no hint in the argument why it can't be so).

Is this a correct approach ? I don't want to arrive at a correct answer with wrong logic.

Thanks.

Posted from my mobile device


Read the conclusion properly. It states that --

"Orange projects that IF one-fourth of people in these markets buy the eSlate, Orange will sell over 5 million units of the eSlate."

What people said in the surveys is irrelevant to the argument at hand. We are ONLY concerned about one scenario -- WHEN 1/4TH of people in these markets buy the e-Slate -- and the conclusion derived when this is ASSUMED to be true.

This is what the argument says --

No. of people with income > 250,000$ + No. of people who are medical professionals = 20 million.
If 25% of these people purchase the item, Orange will sell over 5 million units.

Option E tries to point a flaw in the reasoning above.
Let us assume that each of these groups have 10 million people. But what if 5 million people both earn >250,000$ and are also medical professionals.

Then, the total number of people will be 15 million people (even though the sum of two terms will be 20 million people).
Then, approximate number of units sold = 15 million * 0.25 = 3.75 million. Destroys the argument.
Current Student
Joined: 14 Nov 2016
Posts: 1174
Own Kudos [?]: 20705 [1]
Given Kudos: 926
Location: Malaysia
Concentration: General Management, Strategy
GMAT 1: 750 Q51 V40 (Online)
GPA: 3.53
Send PM
The Orange corporation is conducting market research in preparation fo [#permalink]
1
Kudos
shagalo wrote:
The Orange Corporation is conducting market research in preparation for the launch of its new device, the 3-D eSlate. Thus far, in Orange's market research, two groups have emerged as likely buyers of the eSlate: medical professionals and people making more than $250,000 a year. Since the number of medical professionals in the target market plus the number of people making more than $250,000 a year in the target market is over 20 million people, Orange projects that if one-fourth of people in these target markets buy the eSlate, Orange will sell over 5 million units of the eSlate.

Which of the following, if true, would most weaken the author's conclusion?

(A)  In surveys, less than 5% of medical professionals in the target market said they would buy the eSlate.
(B)  The eSlate has many more uses for education professionals than for medical professionals.
(C)  Previous projections from the Orange Corporation have generally been too optimistic.
(D)  Many medical professionals make more than $250,000 a year.
(E)  People who make more than $250,000 a year buy more electronic devices than people who make less than $250,000 a year


HINT: An argument asking you to weaken a conclusion is generally asking you to identify the assumptions and then select an answer that says that one of those assumptions is not true.

In order to do this, you must diagram the argument, and be able to identify the conclusion you are trying to weaken.

The first sentence of the argument is just giving background information and doesn't seem to really be part of the argument. The premises in the argument seem to go as follows (use “MPs” for “medical professionals” and “250Ks” for “people making more than $250,000 a year”):

eSlate likely buyers: MPs and 250KsMPs + 250Ks = over 20 million
IF 1/4 of those buy the eSlate, THEN over 5 million will buy

Is there a problem with this logic?

Keep in mind that the argument DOES NOT say that one-fourth of the target market WILL buy the eSlate. It only says that IF they did buy it, more than 5 million sales will result.

Therefore, some of the WRONG answers are likely to try to prove that one-fourth of the target market will not buy the eSlate. You don't care! The argument will NOT be weakened by that assertion. The “if” in the argument is very important! (For instance, the argument, “IF America became a monarchy, it would be likely to allow female monarchs” is not weakened by the assertion that America is very unlikely to become a monarchy.)

Examine the premises of this argument as a little math problem. Two groups add up to over 20 million people. If one-fourth of the people in those groups buy a product, over 5 million products will be sold. What's wrong with that?
Intern
Intern
Joined: 22 Jul 2018
Posts: 5
Own Kudos [?]: 7 [0]
Given Kudos: 7
Send PM
Re: The Orange corporation is conducting market research in preparation fo [#permalink]
Lovely question. If people get stuck, it is between options A and D. Option A if true, can still have 5mil+ sales because question does not give split up of people that are Med and "Income over 250000".

D conclusively weakens the argument.

Posted from my mobile device
User avatar
Non-Human User
Joined: 01 Oct 2013
Posts: 17206
Own Kudos [?]: 848 [0]
Given Kudos: 0
Send PM
Re: The Orange corporation is conducting market research in preparation fo [#permalink]
Hello from the GMAT Club VerbalBot!

Thanks to another GMAT Club member, I have just discovered this valuable topic, yet it had no discussion for over a year. I am now bumping it up - doing my job. I think you may find it valuable (esp those replies with Kudos).

Want to see all other topics I dig out? Follow me (click follow button on profile). You will receive a summary of all topics I bump in your profile area as well as via email.
GMAT Club Bot
Re: The Orange corporation is conducting market research in preparation fo [#permalink]
Moderators:
GMAT Club Verbal Expert
6917 posts
GMAT Club Verbal Expert
238 posts
CR Forum Moderator
832 posts

Powered by phpBB © phpBB Group | Emoji artwork provided by EmojiOne